LSAT 15 – Section 3 – Question 25

You need a full course to see this video. Enroll now and get started in less than a minute.

Target time: 0:57

This is question data from the 7Sage LSAT Scorer. You can score your LSATs, track your results, and analyze your performance with pretty charts and vital statistics - all with a Free Account ← sign up in less than 10 seconds

Question
QuickView
Type Tags Answer
Choices
Curve Question
Difficulty
Psg/Game/S
Difficulty
Explanation
PT15 S3 Q25
+LR
Weaken +Weak
A
3%
158
B
2%
151
C
9%
159
D
85%
165
E
3%
162
141
150
158
+Medium 147.322 +SubsectionMedium
This page shows a recording of a live class. We're working hard to create our standard, concise explanation videos for the questions in this PrepTest. Thank you for your patience!

This is a weakening question, since the question stem demands: Which one of the following, if true, most seriously undermines the conclusion that change in the direction of the Earth’s magnetic field happened very slowly?

The stimulus begins with the conditional indicator when, and tells us that whenever lava solidifies, it necessarily magnetizes in the direction that the earth’s magnetic field points at the moment of solidification. Interesting! The next sentence gives us some more information about this event, namely that lava flows from different volcanos that erupted at different points over the past several million years are magnetized in different directions. From this evidence, the next sentence concludes that the direction’s of the earth’s magnetic field has changed over time. Seems like a fairly reasonable conclusion! The next sentence begins with the support indicator since, and whenever we see a sentence that takes the form “since X, Y”, we should expect X to be the final premise introduced before the conclusion Y. That’s exactly what we get here. The final premise tells us that lava flows that are separated by only a few thousand years have very similar magnetization directions. Based on this and our sub-conclusion from the last sentence, the stimulus ends with the main conclusion that the direction of the Earth’s magnetic field changes gradually. Our job is to select the answer choice which most weakens this conclusion; we want something that suggests non-gradual change. Let’s anchor ourselves in the argument, and consider the implications of the answer choices:

Answer Choice (A) This answer might be tempting because the movement of the liquids is described as chaotic, but there’s no reason why a gradual change couldn’t have a chaotic cause.

Answer Choice (B) If anything this answer would suggest gradual change, since we have not detected any real change during the short period we have been monitoring the field.

Answer Choice (C) The problem with this answer is that we don’t know what a complete reversal means in this case; maybe it was a very small change. And even if it was a significant change, it occurred over a long time period (a few million years), so it could still be entirely consistent with gradual change.

Correct Answer Choice (D) This answer does what C fails to; it introduces a significant change in a short timespan. A significant change in the span of weeks is completely inconsistent with a theory of gradual change over thousands of years.

Answer Choice (E) This answer is just entirely irrelevant to the gradual change theory we want to undermine. Who cares how long some lava flows will take to solidify, we want to know whether the magnetic field the point to is changing rapidly!

Take PrepTest

Review Results

Leave a Reply